HELP!!!!! meeeeeejejsjss

HELP!!!!! Meeeeeejejsjss

Answers

Answer 1

Answer:

[tex]3600[/tex]

Step-by-step explanation:

One is asked to find the cost of a vacation when given the following information;

Travel: 150

Hotel: 50 per day

Spending money: 250

One is asked to calculate the cost of the vacation for (4) people over the course of (7) days. In order to solve this problem, one must make a few assumptions.

- Each cost is per person, therefore one will have to multiply the cost by the number of people.

- The travel is per every one time, thus one will have to multiply it by (2) to account for the cost to travel back home.

- Everyone stays in their own hotel room, therefore, one must multiply the hotel cost by the number of people

- The spending money is for the entire vacation and not per day.

With these assumptions, one can form the following equaton;

x = number of people = 4

y = number of days = 7

[tex]travel\ cost= (2(x(150))\\\\hotel\ cost= (y(x(50))\\\\spending\ money= (x(250))\\\\total\ cost= (travel\ cost)+(hotel\ cost) + (spending\ money)[/tex]

Substitute,

[tex]total\ cost= (travel\ cost)+(hotel\ cost) + (spending\ money)[/tex]

[tex]total\ cost= 2(4(150))+(7(4(50))+(4(250))[/tex]

Simplify,

[tex]total\ cost= 2(4(150))+(7(4(50))+(4(250))[/tex]

[tex]total\ cost= 2(600)+(7(4(50))+(4(250))\\=2(600)+7(200)+4(250)\\=2(600)+7(200)+1000\\=2(600)+1400+1000\\=1200+1400+1000\\=2600+1000\\=3600[/tex]


Related Questions

14.8 = n minus 0.3
n = negative 15.1
n = negative 14.5
n = 14.5
n = 15.1

Answers

Hello!

14.8 = n - 0.3

n - 0.3 = 14.8

n = 14.8 + 0.3

n = 15.1 value of n

Good luck! :)

Answer:

D: n = 15.1

Step-by-step explanation:

14.8 = n minus 0.3

14.8= n - 0.3

14.8    - is +
+
0.3
=
15.1
:D if u need help with more just ask!
I love this kind of math.

which of the following statements must be true, given that ΔABC≅ΔXYZ, and the measure of ∠C is 32°

Answers

[tex]\huge\boxed{\boxed{\underline{\textsf{\textbf{Answer}}}}}[/tex]

Given,

ΔABC ≅ ΔXYZ

If these 2 triangles are congruent with each other then,

∠ A = ∠ X [tex]\boxed{\bf{Corresponding \ parts \ of \ congruent \ triangles}}[/tex]

∠ B = ∠ Y [tex]\boxed{\bf{Corresponding \ parts \ of \ congruent \ triangles}}[/tex]

∠ C = ∠ Z [tex]\boxed{\bf{Corresponding \ parts \ of \ congruent \ triangles}}[/tex]

Now,

We saw that ∠ C = ∠ Z.

⟹ So, if ∠ C = 32°, then even ∠ Z will be equal to 32°. [tex]\boxed{\sf{Equal \ angles \ have \ equal \ measurements}}[/tex]

ᶛɲƧཡэʀ ↦ C. m X = 32°

ʰᵒᵖᵉ ⁱᵗ ʰᵉˡᵖˢ

꧁❣ ʀᴀɪɴʙᴏᴡˢᵃˡᵗ2²2² ࿐

which values are soloutions to the inequality -3x - 4 < 2 ? check all of the boxes that apply

Answers

Given:

The inequality is:

[tex]-3x-4<2[/tex]

To find:

The values that are solutions to the given inequality.

Solution:

We have,

[tex]-3x-4<2[/tex]

Adding 4 on both sides, we get

[tex]-3x-4+4<2+4[/tex]

[tex]-3x<6[/tex]

Divide both sides by -3 and change the inequality sign because -3 is a negative value.

[tex]\dfrac{-3x}{-3}>\dfrac{6}{-3}[/tex]

[tex]x>-2[/tex]

Therefore, all the real values greater than -2 are the solutions to the given inequality.

Find the measure of the angle indicated.
1
2
4
3
151°
6
8
7
The measure of angle 7 is

Answers

............................

...............................................................................................................................................................................................................................................................................................................

[tex]\text{Solve for 'x'.}\\\\x^2-25=0\\\\\text{Thank you.}[/tex]

Answers

Hi there!  

»»————- ★ ————-««

I believe your answer is:  

[tex]x = \pm 5[/tex]

»»————- ★ ————-««  

Here’s why:  

⸻⸻⸻⸻

[tex]\boxed{\text{Solving for 'x'...}}\\\\x^2-25 = 0\\-------------\\\rightarrow x^2 -25 + 25 = 0 + 25\\\\\rightarrow x^2 = 25\\\\\rightarrow \sqrt{x^2}=\sqrt{25}\\\\\rightarrow \boxed{x = \pm 5}[/tex]

⸻⸻⸻⸻

»»————- ★ ————-««  

Hope this helps you. I apologize if it’s incorrect.  

Answer:

[tex]x = 5 \: \: \: or \: \: \: x = - 5[/tex]

Step-by-step explanation:

[tex] {x}^{2} - 25 = 0 \\ {x}^{2} - {5}^{2} \\ ( x - 5)(x + 5) = 0 \\ \\ x - 5 = 0 \\ x = 5 \\ or \\ x + 5 = 0 \\ x = - 5[/tex]

Question 1 (5 points)
Determine the value of x.
3
3V2
6
3V3

Answers

Answer:

Step-by-step explanation:

John finds that the sum of two numbers is 24 and their difference is one sixth of the sum. Find the smallest number between the two numbers

Answers

Answer:

The smallest number is 10

Step-by-step explanation:

x+y=24---equation 1

x-y=¹/6×24=>x-y=4---equation 2

Add both equations

2x=28

x=14

put x=14 into equation 1

14+y=24

y=24-14=10

Tom and 29 friends (30 total) are to sit in three rows of 10 at a movie theatre. They madea rule that Within each row, they must sit in order of tallest to shortest with the tallestperson on the left. Given that there are no two people with the same height and there areno restrictions on where a person must sit, how many different seating arrangements arepossible

Answers

Answer:

The answer is "6000".

Step-by-step explanation:

It seems to be a total of 30 buddies there. Every column has 10 seats so that the 10 pals are now in a row. Of all the other 20 buddies, 10 are on the following row. And we have ten friends remaining and that they are sitting in the next row.

Therefore the possibility of sitting is:

[tex]30 \times 20 \times 10 = 6000[/tex]

In ΔEFG, the measure of ∠G=90°, GF = 33, FE = 65, and EG = 56. What ratio represents the sine of ∠F?

Answers

Answer:  56/65

Work Shown:

sin(angle) = opposite/hypotenuse

sin(F) = EG/FE

sin(F) = 56/65

Refer to the diagram below.

Which of the following is the constant ratio of the relation shown in the table?

Answers

There is no constant ratio

Answer:

hello!

where are you from ?

Step-by-step explanation:

option 4 is correct ...there is no constant ratio.

Find the number of degrees in the measure of angle x

Answers

Answer: x = 82°

Step-by-step explanation:

The angle on the other side of 108° can be calculated as 180° - 108° = 72°

All angles within a triangle add up to 180°, so the x-value can be found as:

x = 180° - 72° - 26° = 82°

Find the value of x that will make A||B.

Please help!

Answers

Answer:

x=30

Step-by-step explanation:

Hi there!

For A to be parallel to B, 5x would be equal to 3x+60. (If they were parallel, these two angles would be alternate exterior angles, which are equal.)

[tex]5x=3x+60[/tex]

Subtract 3x from both sides

[tex]5x-3x=3x+60-3x\\2x=60[/tex]

Divide both sides by 2

[tex]x=30[/tex]

I hope this helps!

Function A and Function B are linear functions. Function A x y – 10 – 14 – 1 – 5 9 5 Function B y=2x+4 Which statement is true?

Answers

Answer:

See explanation

Step-by-step explanation:

Function A is not clear; I will use the following in place of function A

Function A:

[tex]x \to\ 1 |\ 3 |\ 4 |\ 6[/tex]

[tex]y \to -1|\ 3|\ 5|\ 9[/tex]

Function B:

[tex]y = 2x + 4[/tex]

Required

Compare both functions

For linear functions, we often compare the slope and the y intercepts only.

Calculating the slope of function A, we have:

[tex]m = \frac{y_2 - y_1}{x_2 - x_1}[/tex]

Where:

[tex](x_1,y_1) = (1,-1)[/tex]

[tex](x_2,y_2) = (3,3)[/tex]

So, we have:

[tex]m = \frac{3 - -1}{3 - 1}[/tex]

[tex]m = \frac{4}{2}[/tex]

[tex]m = 2[/tex]

To calculate the y intercept, we set [tex]x = 0[/tex], then solve for y

i.e.[tex](x,y) = (0,y)[/tex]

Using the slope formula, we have:

[tex]m = \frac{y_2 - y_1}{x_2 - x_1}[/tex]

Where:

[tex]m = 2[/tex]

[tex](x_1,y_1) = (0,y)[/tex]

[tex](x_2,y_2) = (3,3)[/tex]

So, we have:

[tex]2 = \frac{3 - y}{3 - 0}[/tex]

[tex]2 = \frac{3 - y}{3}[/tex]

Multiply by 3

[tex]6 = 3 - y[/tex]

Collect like terms

[tex]y = 3 - 6[/tex]

[tex]y = -3[/tex]

So, for function A:

[tex]m = 2[/tex] -- slope

[tex]y = -3[/tex] --- y intercept

For function B

[tex]y = 2x + 4[/tex]

A linear function is represented as:

[tex]y = mx + b[/tex]

By comparison

[tex]m = 2[/tex] --- slope

[tex]b = 4[/tex] --- y intercept

By comparing the results of both functions, we have the following conclusion:

Functions A and B have the same slope (i.e. 2)

Function B has a greater y intercept (i.e. 4)

Word problem One of the citizens has 97 silver coins. How many bronze coins would it take to equal this amount

Answers

Given: Given that a citizen have 97 silver coins.

To find : Here we need to find that how many bronze coins would it take to equal this amount.

Solution: We know, 1 silver coin=10 bronze coin

So, 97 silver coin=10×97 bronze coin

=970 bronze coin

Therefore, 970 bronze coins would it take to equal this amount.

Find the product: 3/4 x 2/3. What's the product? 5/7, 6/12, 5/12, 6/7. What is it please help me!!!!

Answers

Answer:

1/2

Step-by-step explanation:

3/4 x 2/3

= 1/2

what is the length of segment LM?

Answers

SolutioN :-

Here we are provided with a diagram of a triangle. We need to find out the length of the segment LM . As we can see that ,

∆ KNL ≈ MNL , [ By AAS ]

Therefore ,

KN = MN ( by cpct )

⇒ KN = MN

⇒ 14x - 3 = 25

⇒ 14x = 25 + 3

⇒ 14x = 28

x = 2

Put this x = 2 in LM :-

⇒ LM = 9x + 5

⇒ LM = 9*2 + 5

⇒ LM = 18 + 5

LM = 23

Hence the required answer is 23 .

I need help ASAP !!!

Answers

The answer is (-1,12)
Center (-1, 12)
Radius 5
Diameter 10
Area 25 π
Circumference 10 π

write an expression for 15 divided by a number
show your work ​

Answers

Answer:

15/X

Step-by-step explanation:

a number divided by 15

15/X

I think it’s 15/x = x<5 maybe

What is the value of x |-16|

Answers

Answer:

16

Step-by-step explanation:

|-16| = the distance between -16 and 0
Hence, the answer is 16
16x I believe .........

is AC greater than, less than, or equal to BC? explain your reasoning

Answers

Answer:

AC is greater than BC

Step-by-step explanation:

First, we know that the angle of a straight line is 180°, so angle B as a whole is equal to 180 degrees. Therefore, angle YBC + angle ABC = 180 degrees. As angle YBC is a right angle, signified by the small square on the angle, it is 90 degrees. Therefore,

90 degrees + angle ABC = 180 degrees

subtract 90 degrees from both sides to isolate angle ABC

angle ABC = 90 degrees

Therefore, as angle ABC is equal to 90 degrees, and a right angle is 90 degrees, triangle ABC has a right angle, making it a right triangle.

In a right triangle, using the Pythagorean Theorem, the square of the side opposite the right angle is equal to the sum of the squares of the other side. Since side AC is opposite the right angle, we can say that

AC² = AB² + BC²

As the length of a side has to be greater than 0, we can say that

AC² = AB² + BC²

AB² > 0

AC² > BC²

square root both sides

AC > BC

Therefore, AC is greater than BC

Is (0,0) a solution of the graphed inequality?
Choose 1 answer:
Yes
No

Answers

Answer:

no..........................

Jeff and Cameron are arguing about which one of them is faster. Jeff says "I can run 777 kilometers per hour!" and Cameron says "I can run 100100100 meters per minute!

Answers

Answer:

Jeff is moving faster.

Step-by-step explanation:

To compare two speeds, first we make them in one unit.

We know that,

1 km/h = 0.2777 m/s

7 km/h = 1.94 m/s

Jeff can run at a speed of 7 km/h i.e. 1.94 m/s while Cameron can run with a speed of 10 m/min or 0.167 m/s.

On comparing 1.94 m/s and 0.167 m/s, we found that Jeff is moving with more speed.

So, Jeff is faster.

A square pyramid is inscribed in a rectangular prism. A cone is inscribed in a cylinder. The pyramid and the cone have the same volume. Part of the volume of the rectangular prism, 1 V 1 , is not taken up by the square pyramid. Part of the volume of the cylinder, 2 V 2 , is not taken up by the cone. What is the relationship of these two volumes, 1 V 1 and 2 V 2 ?

Answers

Answer:

V₂ = V₁

Step-by-step explanation:

Let the height of the rectangular prism = h

Let s represent the side length of the base of the square prism, we have;

The volume of the prism, [tex]V_{prism}[/tex] = s²·h

The volume of the square pyramid, [tex]V_{pyramid}[/tex] = (1/3)·s²·h

∴ V₁ = The area not taken up by the square pyramid = [tex]V_{prism}[/tex] - [tex]V_{pyramid}[/tex]

∴ V₁ = s²·h - (1/3)·s²·h = (2/3)·s²·h

Similarly, for the cylinder, we have;

Let h represent the height of the cylinder

Let r represent the radius of the base of the cone, we have;

Therefore;

The volume of the cylinder, [tex]V_{cylinder}[/tex] = π·r²·h

The volume of the cone, [tex]V_{cone}[/tex] = (1/3)·π·r²·h

∴ V₂ = π·r²·h - (1/3)·π·r²·h = (2/3)·π·r²·h

V₂ = (2/3)·π·r²·h

[tex]V_{cone}[/tex] = [tex]V_{pyramid}[/tex]

Therefore;

(1/3)·π·r²·h = (1/3)·s²·h

∴ π·r² = s²

Therefore, V₂ = (2/3)·π·r²·h = V₂ = (2/3)·s²·h = V₁

V₂ = V₁.

A 4 metre ladder is placed against a vertical wall.
The base of the ladder is 1.5 metres from the base of the wall.

Answers

Answer:

Original position: base is 1.5 meters away from the wall and the vertical distance from the top end to the ground let it be y and length of the ladder be L.

Step-by-step explanation:

By pythagorean theorem, L^2=y^2+(1.5)^2=y^2+2.25 Eq1.

Final position: base is 2 meters away, and the vertical distance from top end to the ground is y - 0.25 because it falls down the wall 0.25 meters and length of the ladder is also L.

By pythagorean theorem, L^2=(y -0.25)^2+(2)^2=y^2–0.5y+ 0.0625+4=y^2–0.5y+4.0625 Eq 2.

Equating both Eq 1 and Eq 2: y^2+2.25=y^2–0.5y+4.0625

y^2-y^2+0.5y+2.25–4.0625=0

0.5y- 1.8125=0

0.5y=1.8125

y=1.8125/0.5= 3.625

Using Eq 1: L^2=(3.625)^2+2.25=15.390625, L=(15.390625)^1/2= 3.92 meters length of ladder

Using Eq 2: L^2=(3.625)^2–0.5(3.625)+4.0625

L^2=13.140625–0.90625+4.0615=15.390625

L= (15.390625)^1/2= 3.92 meters length of ladder

hope it helps...

correct me if I'm wrong...

Given a line segment that contains the points A,B, & C in order, if AB = 2x - 2, and BC = 2x + 10, and AC = 32, find x.
Select one: a. 6
b. 24
c. 8
d. - 4

Answers

Answer:

a. 6

Step-by-step explanation:

AB +BC =AC

2x-2+2x+10=32

4x+8=32

4x=32-8

4x=24

x=24/4

x=6

280L of water consumed my 7 people. water consumed by 50 people =___L

Answers

Step-by-step explanation:

7 people = 280 liters

1 p = 40 liters

50 p = 40 x 50

50 p = 2000 liters

hope it helps.

Also, I think that Brainly is an awesome app, but there's an app which is doing great work for me in maths, named Gauthmath. I will suggest it. Video concepts and answers from real tutors.

Find the area enclosed by the figure.

Answers

Answer:

894

Step-by-step explanation:

this can be "split" into 3 rectangles.

their areas can be easily calculated. and then we simply sum them all up for the answer.

rectangle 1 on the top

R1 = 5×9 = 45

rectangle 2 in the middle

R2 = (19+5)×(35-9-15) = 24×11 = 264

rectangle 3 at the bottom

R3 = 39×15 = 585

and all together

45+264+585 = 894

simplify the following.(2x-y)(x+2y)​

Answers

Answer:

[tex]=2x^2+3xy-2y^2[/tex]

Step-by-step explanation:

When given the following problem;

[tex](2x-y)(x+2y)[/tex]

Distribute, multiply every number in one of the parenthesis by every number in the other;

[tex](2x-y)(x+2y)\\=(2x)(x)+(2x)(2y)+(-y)(x)+(2y)(-y)[/tex]

Simplify,

[tex]=(2x)(x)+(2x)(2y)+(-y)(x)+(2y)(-y)\\=2x^2+4xy-xy-2y^2\\=2x^2+3xy-2y^2[/tex]

Therefore, the final answer is;

[tex]=2x^2+3xy-2y^2[/tex]

Work out the area of the shaded shape.

Answers

Answer:

65 m²

Step-by-step explanation:

Area 1 :-

A = 3m * 9m A = 27 m²

Area 2 :-

A = (12-3-2) m * (9 - 5) m A = 7m * 4 mA = 28 m²

Area 3 :-

Area = 2m * 5 m A = 10 m²

Total Area :-

A = ( 27 + 28 + 10 ) m²A = 65 m²

( t^15/27x^9 ) ^-2/3

Please solve this
it is fraction with a negative fraction exponent

Answers

Answer:

Step-by-step explanation:

Exponent laws:

[tex](a^{m})^{n}=a^{m*n}\\\\a^{-m}=\frac{1}{a^{m}}[/tex]

[tex](t^{\frac{15}{27}}x^{9}})^{\frac{-2}{3}}= \ t^{\frac{15}{27}*\frac{-2}{3}}*x^{9*\frac{-2}{3}}\\\\\\= t^{\frac{5}{9}*\frac{-2}{3}}*x^{3*-2}\\\\=t^{\frac{-10}{27}}x^{-6}\\\\=\frac{1}{t^{\frac{10}{27}}x^{6}}[/tex]

Other Questions
Which is equivalent to 106 please help (picture) its due in an hour How did television contribute to the lack of support for the Vietnam War in the U.S.?A. For the first time, people back home could really see the brutality of war firsthand.B. The American public cared more about entertainment than containing Communism.C. Most reporters sided with the Vietnamese and portrayed American soldiers as murderers.D. None of these choices are correct. A child is suffering from fever but the doctor cannot immediately pinpoint the ailment on the basis of this one symptom. Explain why. Also mention another two such general symptoms. PLS HURRY Make predictions about the subject and likely point of view of the following magazine articles.Saudi Arabia: A Country Without a Constitution Edge 2020-2021 ANSWER AS FAST A POSSIBLE PLS! 156-3^2x5-8^2 pls help geometry! Surface area In a simple machine the energy input is 120J if the efficiency of the machine is 80% calculate the energy output Any Netflix shows i can watch that are actually interesting or sad? Bow-Wow Company manufactures a product with a unit variable cost of $50 and a unit sales price of $88. Fixed manufacturing costs were $240,000 when 10,000 units were produced and sold. The company has a one-time opportunity to sell an additional 1,000 units at $70 each in a foreign market which would not affect its present sales. If the company has sufficient (excess) capacity to produce the additional units, acceptance of the special order would affect net income as follows:a. Income would decrease by $4,000. b. Income would increase by $4,000. c. Income would increase by $70,000. d. Income would increase by $20,000. During starch test why is leaf boiled in ethanol?A) to kill its ProtoplasmB) to soften itC) to extract its chlorophyllD) to retain its starch Which of the following shows the correct energy flow during aerobic respiration?A. Lipids, Glucose, ATP, cellsB. ATP, Glucose, cellsC. Carbohydrates, Glucose, ATP, cellsD. Carbohydrates, Glucose, Lactic acid, ATP name a device that converts mechanical energy into electrical energy. writing about your shopping habits Use the pressure meter to read the pressure in Fluid A at the bottom of the tank. Do not move the pressure meter. Switch to Fluid B and read the pressure in fluid B. Based on the two readings, compare the density of fluid B to the density of fluid A. Which statement is correct? If you could travel 900 meters in 55 seconds, what is your speed?(Answer in details=brainliest) hey , would you like to talk? Help please. I need the answer 3. Elige correcto o incorrecto. (24 points /2 points each correct sentence)Choose correct or incorrect for each sentence. a. Pedro es un peridico. correcto incorrectob. T coma chocolate. correcto incorrectoc. Mi nombre Ahmad. correcto incorrectod. Vivo en Australia. correcto incorrectoe. Ella es de sueca. correcto incorrectof. Cmo se dice hola en rabe?correcto incorrectog. Laura trabaja una tienda. correcto incorrectoh. La nia es italiano. correcto incorrectoi. Juan tiene doce ao. correcto incorrectoj. Como por la tarde. correcto incorrectok. Qu idiomas hablas?correcto incorrectol. Yo estudio de finanzas. correcto incorre diffrences between pine tree and mango tree